Show that the following sequence converges. Please Critique my proof.












6












$begingroup$


The problem is as follows:




Let ${a_n}$ be a sequence of nonnegative numbers such that
$$
a_{n+1}leq a_n+frac{(-1)^n}{n}.
$$

Show that $a_n$ converges.




My (wrong) proof:



Notice that
$$
|a_{n+1}-a_n|leq left|frac{(-1)^n}{n}right|leqfrac{1}{n}
$$

and since it is known that $frac{1}{n}rightarrow 0$ as $nrightarrow infty$. We see that we can arbitarily bound, $|a_{n+1}-a_n|$. Thus, $a_n$ converges.



My question:
This is a question from a comprehensive exam I found and am using to review.



Should I argue that we should select $N$ so that $n>N$ implies $left|frac{1}{n}right|<epsilon$ as well?



Notes: Currently working on the proof.










share|cite|improve this question











$endgroup$








  • 4




    $begingroup$
    Your proof is not correct. Your arguments would also work for $a_n = sum_{i=1}^n frac 1 i$, which does not converge.
    $endgroup$
    – Falrach
    2 hours ago






  • 1




    $begingroup$
    Note that you not only need to bound $left| a_{n+1} - a_n right|$ arbitrarily small, but also $left| a_{m} - a_n right|$ for all $m,n geq N$ (where $N$ can be chosen according to the bound).
    $endgroup$
    – Maximilian Janisch
    44 mins ago


















6












$begingroup$


The problem is as follows:




Let ${a_n}$ be a sequence of nonnegative numbers such that
$$
a_{n+1}leq a_n+frac{(-1)^n}{n}.
$$

Show that $a_n$ converges.




My (wrong) proof:



Notice that
$$
|a_{n+1}-a_n|leq left|frac{(-1)^n}{n}right|leqfrac{1}{n}
$$

and since it is known that $frac{1}{n}rightarrow 0$ as $nrightarrow infty$. We see that we can arbitarily bound, $|a_{n+1}-a_n|$. Thus, $a_n$ converges.



My question:
This is a question from a comprehensive exam I found and am using to review.



Should I argue that we should select $N$ so that $n>N$ implies $left|frac{1}{n}right|<epsilon$ as well?



Notes: Currently working on the proof.










share|cite|improve this question











$endgroup$








  • 4




    $begingroup$
    Your proof is not correct. Your arguments would also work for $a_n = sum_{i=1}^n frac 1 i$, which does not converge.
    $endgroup$
    – Falrach
    2 hours ago






  • 1




    $begingroup$
    Note that you not only need to bound $left| a_{n+1} - a_n right|$ arbitrarily small, but also $left| a_{m} - a_n right|$ for all $m,n geq N$ (where $N$ can be chosen according to the bound).
    $endgroup$
    – Maximilian Janisch
    44 mins ago
















6












6








6


1



$begingroup$


The problem is as follows:




Let ${a_n}$ be a sequence of nonnegative numbers such that
$$
a_{n+1}leq a_n+frac{(-1)^n}{n}.
$$

Show that $a_n$ converges.




My (wrong) proof:



Notice that
$$
|a_{n+1}-a_n|leq left|frac{(-1)^n}{n}right|leqfrac{1}{n}
$$

and since it is known that $frac{1}{n}rightarrow 0$ as $nrightarrow infty$. We see that we can arbitarily bound, $|a_{n+1}-a_n|$. Thus, $a_n$ converges.



My question:
This is a question from a comprehensive exam I found and am using to review.



Should I argue that we should select $N$ so that $n>N$ implies $left|frac{1}{n}right|<epsilon$ as well?



Notes: Currently working on the proof.










share|cite|improve this question











$endgroup$




The problem is as follows:




Let ${a_n}$ be a sequence of nonnegative numbers such that
$$
a_{n+1}leq a_n+frac{(-1)^n}{n}.
$$

Show that $a_n$ converges.




My (wrong) proof:



Notice that
$$
|a_{n+1}-a_n|leq left|frac{(-1)^n}{n}right|leqfrac{1}{n}
$$

and since it is known that $frac{1}{n}rightarrow 0$ as $nrightarrow infty$. We see that we can arbitarily bound, $|a_{n+1}-a_n|$. Thus, $a_n$ converges.



My question:
This is a question from a comprehensive exam I found and am using to review.



Should I argue that we should select $N$ so that $n>N$ implies $left|frac{1}{n}right|<epsilon$ as well?



Notes: Currently working on the proof.







real-analysis sequences-and-series convergence fake-proofs






share|cite|improve this question















share|cite|improve this question













share|cite|improve this question




share|cite|improve this question








edited 43 mins ago









GNUSupporter 8964民主女神 地下教會

13.9k72650




13.9k72650










asked 2 hours ago









DarelDarel

1049




1049








  • 4




    $begingroup$
    Your proof is not correct. Your arguments would also work for $a_n = sum_{i=1}^n frac 1 i$, which does not converge.
    $endgroup$
    – Falrach
    2 hours ago






  • 1




    $begingroup$
    Note that you not only need to bound $left| a_{n+1} - a_n right|$ arbitrarily small, but also $left| a_{m} - a_n right|$ for all $m,n geq N$ (where $N$ can be chosen according to the bound).
    $endgroup$
    – Maximilian Janisch
    44 mins ago
















  • 4




    $begingroup$
    Your proof is not correct. Your arguments would also work for $a_n = sum_{i=1}^n frac 1 i$, which does not converge.
    $endgroup$
    – Falrach
    2 hours ago






  • 1




    $begingroup$
    Note that you not only need to bound $left| a_{n+1} - a_n right|$ arbitrarily small, but also $left| a_{m} - a_n right|$ for all $m,n geq N$ (where $N$ can be chosen according to the bound).
    $endgroup$
    – Maximilian Janisch
    44 mins ago










4




4




$begingroup$
Your proof is not correct. Your arguments would also work for $a_n = sum_{i=1}^n frac 1 i$, which does not converge.
$endgroup$
– Falrach
2 hours ago




$begingroup$
Your proof is not correct. Your arguments would also work for $a_n = sum_{i=1}^n frac 1 i$, which does not converge.
$endgroup$
– Falrach
2 hours ago




1




1




$begingroup$
Note that you not only need to bound $left| a_{n+1} - a_n right|$ arbitrarily small, but also $left| a_{m} - a_n right|$ for all $m,n geq N$ (where $N$ can be chosen according to the bound).
$endgroup$
– Maximilian Janisch
44 mins ago






$begingroup$
Note that you not only need to bound $left| a_{n+1} - a_n right|$ arbitrarily small, but also $left| a_{m} - a_n right|$ for all $m,n geq N$ (where $N$ can be chosen according to the bound).
$endgroup$
– Maximilian Janisch
44 mins ago












2 Answers
2






active

oldest

votes


















3












$begingroup$

Consider $b_n = a_n + sum_{k=1}^{n-1} frac{(-1)^{k-1}}{k}$. Then



$$ b_{n+1}
= a_{n+1} + sum_{k=1}^{n} frac{(-1)^{k-1}}{k}
leq a_n + frac{(-1)^n}{n} + sum_{k=1}^{n} frac{(-1)^{k-1}}{k}
= b_n, $$



which shows that $(b_n)$ is non-increasing. Moreover, since $sum_{k=1}^{infty} frac{(-1)^{k-1}}{k}$ converges by alternating series test and $(a_n)$ is non-negative, it follows that $(b_n)$ is bounded from below. Therefore $(b_n)$ converges, and so, $(a_n)$ converges as well.






share|cite|improve this answer









$endgroup$









  • 2




    $begingroup$
    Thank you, that's neat! One might add that this argument always works for lower-bounded $(a_n)$ with $a_{n+1}le a_n+c_n$ for some summable $(c_n)$ by setting $b_n=a_n-sum_{k=1}^{n-1}c_k$.
    $endgroup$
    – Mars Plastic
    29 mins ago





















0












$begingroup$

Use $$sum_{k=1}^n(a_{k+1}-a_k)=a_{n+1}-a_1leqsum_{k=1}^nfrac{(-1)^k}{k}rightarrow -ln2$$






share|cite|improve this answer











$endgroup$









  • 1




    $begingroup$
    Be careful! The assumption is only an inequality.
    $endgroup$
    – Mars Plastic
    2 hours ago






  • 1




    $begingroup$
    @Mars Plastic I see. It was typo.
    $endgroup$
    – Michael Rozenberg
    2 hours ago










  • $begingroup$
    That shows that $(a_n)$ is bounded above, but why is it convergent?
    $endgroup$
    – Martin R
    38 mins ago











Your Answer





StackExchange.ifUsing("editor", function () {
return StackExchange.using("mathjaxEditing", function () {
StackExchange.MarkdownEditor.creationCallbacks.add(function (editor, postfix) {
StackExchange.mathjaxEditing.prepareWmdForMathJax(editor, postfix, [["$", "$"], ["\\(","\\)"]]);
});
});
}, "mathjax-editing");

StackExchange.ready(function() {
var channelOptions = {
tags: "".split(" "),
id: "69"
};
initTagRenderer("".split(" "), "".split(" "), channelOptions);

StackExchange.using("externalEditor", function() {
// Have to fire editor after snippets, if snippets enabled
if (StackExchange.settings.snippets.snippetsEnabled) {
StackExchange.using("snippets", function() {
createEditor();
});
}
else {
createEditor();
}
});

function createEditor() {
StackExchange.prepareEditor({
heartbeatType: 'answer',
autoActivateHeartbeat: false,
convertImagesToLinks: true,
noModals: true,
showLowRepImageUploadWarning: true,
reputationToPostImages: 10,
bindNavPrevention: true,
postfix: "",
imageUploader: {
brandingHtml: "Powered by u003ca class="icon-imgur-white" href="https://imgur.com/"u003eu003c/au003e",
contentPolicyHtml: "User contributions licensed under u003ca href="https://creativecommons.org/licenses/by-sa/3.0/"u003ecc by-sa 3.0 with attribution requiredu003c/au003e u003ca href="https://stackoverflow.com/legal/content-policy"u003e(content policy)u003c/au003e",
allowUrls: true
},
noCode: true, onDemand: true,
discardSelector: ".discard-answer"
,immediatelyShowMarkdownHelp:true
});


}
});














draft saved

draft discarded


















StackExchange.ready(
function () {
StackExchange.openid.initPostLogin('.new-post-login', 'https%3a%2f%2fmath.stackexchange.com%2fquestions%2f3131816%2fshow-that-the-following-sequence-converges-please-critique-my-proof%23new-answer', 'question_page');
}
);

Post as a guest















Required, but never shown

























2 Answers
2






active

oldest

votes








2 Answers
2






active

oldest

votes









active

oldest

votes






active

oldest

votes









3












$begingroup$

Consider $b_n = a_n + sum_{k=1}^{n-1} frac{(-1)^{k-1}}{k}$. Then



$$ b_{n+1}
= a_{n+1} + sum_{k=1}^{n} frac{(-1)^{k-1}}{k}
leq a_n + frac{(-1)^n}{n} + sum_{k=1}^{n} frac{(-1)^{k-1}}{k}
= b_n, $$



which shows that $(b_n)$ is non-increasing. Moreover, since $sum_{k=1}^{infty} frac{(-1)^{k-1}}{k}$ converges by alternating series test and $(a_n)$ is non-negative, it follows that $(b_n)$ is bounded from below. Therefore $(b_n)$ converges, and so, $(a_n)$ converges as well.






share|cite|improve this answer









$endgroup$









  • 2




    $begingroup$
    Thank you, that's neat! One might add that this argument always works for lower-bounded $(a_n)$ with $a_{n+1}le a_n+c_n$ for some summable $(c_n)$ by setting $b_n=a_n-sum_{k=1}^{n-1}c_k$.
    $endgroup$
    – Mars Plastic
    29 mins ago


















3












$begingroup$

Consider $b_n = a_n + sum_{k=1}^{n-1} frac{(-1)^{k-1}}{k}$. Then



$$ b_{n+1}
= a_{n+1} + sum_{k=1}^{n} frac{(-1)^{k-1}}{k}
leq a_n + frac{(-1)^n}{n} + sum_{k=1}^{n} frac{(-1)^{k-1}}{k}
= b_n, $$



which shows that $(b_n)$ is non-increasing. Moreover, since $sum_{k=1}^{infty} frac{(-1)^{k-1}}{k}$ converges by alternating series test and $(a_n)$ is non-negative, it follows that $(b_n)$ is bounded from below. Therefore $(b_n)$ converges, and so, $(a_n)$ converges as well.






share|cite|improve this answer









$endgroup$









  • 2




    $begingroup$
    Thank you, that's neat! One might add that this argument always works for lower-bounded $(a_n)$ with $a_{n+1}le a_n+c_n$ for some summable $(c_n)$ by setting $b_n=a_n-sum_{k=1}^{n-1}c_k$.
    $endgroup$
    – Mars Plastic
    29 mins ago
















3












3








3





$begingroup$

Consider $b_n = a_n + sum_{k=1}^{n-1} frac{(-1)^{k-1}}{k}$. Then



$$ b_{n+1}
= a_{n+1} + sum_{k=1}^{n} frac{(-1)^{k-1}}{k}
leq a_n + frac{(-1)^n}{n} + sum_{k=1}^{n} frac{(-1)^{k-1}}{k}
= b_n, $$



which shows that $(b_n)$ is non-increasing. Moreover, since $sum_{k=1}^{infty} frac{(-1)^{k-1}}{k}$ converges by alternating series test and $(a_n)$ is non-negative, it follows that $(b_n)$ is bounded from below. Therefore $(b_n)$ converges, and so, $(a_n)$ converges as well.






share|cite|improve this answer









$endgroup$



Consider $b_n = a_n + sum_{k=1}^{n-1} frac{(-1)^{k-1}}{k}$. Then



$$ b_{n+1}
= a_{n+1} + sum_{k=1}^{n} frac{(-1)^{k-1}}{k}
leq a_n + frac{(-1)^n}{n} + sum_{k=1}^{n} frac{(-1)^{k-1}}{k}
= b_n, $$



which shows that $(b_n)$ is non-increasing. Moreover, since $sum_{k=1}^{infty} frac{(-1)^{k-1}}{k}$ converges by alternating series test and $(a_n)$ is non-negative, it follows that $(b_n)$ is bounded from below. Therefore $(b_n)$ converges, and so, $(a_n)$ converges as well.







share|cite|improve this answer












share|cite|improve this answer



share|cite|improve this answer










answered 47 mins ago









Sangchul LeeSangchul Lee

95k12170276




95k12170276








  • 2




    $begingroup$
    Thank you, that's neat! One might add that this argument always works for lower-bounded $(a_n)$ with $a_{n+1}le a_n+c_n$ for some summable $(c_n)$ by setting $b_n=a_n-sum_{k=1}^{n-1}c_k$.
    $endgroup$
    – Mars Plastic
    29 mins ago
















  • 2




    $begingroup$
    Thank you, that's neat! One might add that this argument always works for lower-bounded $(a_n)$ with $a_{n+1}le a_n+c_n$ for some summable $(c_n)$ by setting $b_n=a_n-sum_{k=1}^{n-1}c_k$.
    $endgroup$
    – Mars Plastic
    29 mins ago










2




2




$begingroup$
Thank you, that's neat! One might add that this argument always works for lower-bounded $(a_n)$ with $a_{n+1}le a_n+c_n$ for some summable $(c_n)$ by setting $b_n=a_n-sum_{k=1}^{n-1}c_k$.
$endgroup$
– Mars Plastic
29 mins ago






$begingroup$
Thank you, that's neat! One might add that this argument always works for lower-bounded $(a_n)$ with $a_{n+1}le a_n+c_n$ for some summable $(c_n)$ by setting $b_n=a_n-sum_{k=1}^{n-1}c_k$.
$endgroup$
– Mars Plastic
29 mins ago













0












$begingroup$

Use $$sum_{k=1}^n(a_{k+1}-a_k)=a_{n+1}-a_1leqsum_{k=1}^nfrac{(-1)^k}{k}rightarrow -ln2$$






share|cite|improve this answer











$endgroup$









  • 1




    $begingroup$
    Be careful! The assumption is only an inequality.
    $endgroup$
    – Mars Plastic
    2 hours ago






  • 1




    $begingroup$
    @Mars Plastic I see. It was typo.
    $endgroup$
    – Michael Rozenberg
    2 hours ago










  • $begingroup$
    That shows that $(a_n)$ is bounded above, but why is it convergent?
    $endgroup$
    – Martin R
    38 mins ago
















0












$begingroup$

Use $$sum_{k=1}^n(a_{k+1}-a_k)=a_{n+1}-a_1leqsum_{k=1}^nfrac{(-1)^k}{k}rightarrow -ln2$$






share|cite|improve this answer











$endgroup$









  • 1




    $begingroup$
    Be careful! The assumption is only an inequality.
    $endgroup$
    – Mars Plastic
    2 hours ago






  • 1




    $begingroup$
    @Mars Plastic I see. It was typo.
    $endgroup$
    – Michael Rozenberg
    2 hours ago










  • $begingroup$
    That shows that $(a_n)$ is bounded above, but why is it convergent?
    $endgroup$
    – Martin R
    38 mins ago














0












0








0





$begingroup$

Use $$sum_{k=1}^n(a_{k+1}-a_k)=a_{n+1}-a_1leqsum_{k=1}^nfrac{(-1)^k}{k}rightarrow -ln2$$






share|cite|improve this answer











$endgroup$



Use $$sum_{k=1}^n(a_{k+1}-a_k)=a_{n+1}-a_1leqsum_{k=1}^nfrac{(-1)^k}{k}rightarrow -ln2$$







share|cite|improve this answer














share|cite|improve this answer



share|cite|improve this answer








edited 2 hours ago

























answered 2 hours ago









Michael RozenbergMichael Rozenberg

106k1893198




106k1893198








  • 1




    $begingroup$
    Be careful! The assumption is only an inequality.
    $endgroup$
    – Mars Plastic
    2 hours ago






  • 1




    $begingroup$
    @Mars Plastic I see. It was typo.
    $endgroup$
    – Michael Rozenberg
    2 hours ago










  • $begingroup$
    That shows that $(a_n)$ is bounded above, but why is it convergent?
    $endgroup$
    – Martin R
    38 mins ago














  • 1




    $begingroup$
    Be careful! The assumption is only an inequality.
    $endgroup$
    – Mars Plastic
    2 hours ago






  • 1




    $begingroup$
    @Mars Plastic I see. It was typo.
    $endgroup$
    – Michael Rozenberg
    2 hours ago










  • $begingroup$
    That shows that $(a_n)$ is bounded above, but why is it convergent?
    $endgroup$
    – Martin R
    38 mins ago








1




1




$begingroup$
Be careful! The assumption is only an inequality.
$endgroup$
– Mars Plastic
2 hours ago




$begingroup$
Be careful! The assumption is only an inequality.
$endgroup$
– Mars Plastic
2 hours ago




1




1




$begingroup$
@Mars Plastic I see. It was typo.
$endgroup$
– Michael Rozenberg
2 hours ago




$begingroup$
@Mars Plastic I see. It was typo.
$endgroup$
– Michael Rozenberg
2 hours ago












$begingroup$
That shows that $(a_n)$ is bounded above, but why is it convergent?
$endgroup$
– Martin R
38 mins ago




$begingroup$
That shows that $(a_n)$ is bounded above, but why is it convergent?
$endgroup$
– Martin R
38 mins ago


















draft saved

draft discarded




















































Thanks for contributing an answer to Mathematics Stack Exchange!


  • Please be sure to answer the question. Provide details and share your research!

But avoid



  • Asking for help, clarification, or responding to other answers.

  • Making statements based on opinion; back them up with references or personal experience.


Use MathJax to format equations. MathJax reference.


To learn more, see our tips on writing great answers.




draft saved


draft discarded














StackExchange.ready(
function () {
StackExchange.openid.initPostLogin('.new-post-login', 'https%3a%2f%2fmath.stackexchange.com%2fquestions%2f3131816%2fshow-that-the-following-sequence-converges-please-critique-my-proof%23new-answer', 'question_page');
}
);

Post as a guest















Required, but never shown





















































Required, but never shown














Required, but never shown












Required, but never shown







Required, but never shown

































Required, but never shown














Required, but never shown












Required, but never shown







Required, but never shown







Popular posts from this blog

What are all the squawk codes?

What are differences between VBoxVGA, VMSVGA and VBoxSVGA in VirtualBox?

Hudsonelva